2
$\begingroup$

Equation 7 of this paper (Ramazan Türkmen, Zübeyde Ulukök, Inequalities for Singular Values of Positive Semidefinite Block Matrices, International Mathematical Forum, Vol. 6, 2011, no. 31, 1535 - 1545) says that it is well known that, given two complex-valued matrices $A$ and $B$, it holds

$$\sigma_j(A+B) \geq \sigma_j(A)+\lambda_n(B)$$

where $\sigma_j$ is the $j$-th singular value and $\lambda_n$ is the $n$-th eigenvalue

  1. Is this correct? Do we need some hypotheses on $A$, $B$?

  2. Can somebody point me to a reference?

Thank you.

$\endgroup$
7
  • 4
    $\begingroup$ Seems a little tough if $B=-A$. $\endgroup$ Nov 17, 2020 at 16:20
  • $\begingroup$ I see. But in this case $\sigma_j(0)=0\geq \sigma_j(A)-\lambda_1(A)$. I think this is possible. $\endgroup$ Nov 17, 2020 at 16:28
  • 3
    $\begingroup$ No. Take $B$ the identity and $A=-B$. Your inequality reads $0 \geq 2$. $\endgroup$ Nov 17, 2020 at 17:16
  • 4
    $\begingroup$ I think the title of paper gives a hint that the matrices are expected to be positive semi definite. $\endgroup$ Nov 17, 2020 at 19:45
  • 1
    $\begingroup$ That's Weyl inequality when $A$ and $B$ are hermitian. $\endgroup$ Nov 18, 2020 at 0:31

1 Answer 1

0
$\begingroup$

Jumping in many years later, but I believe there's a typo in the paper which originates from Zhang's 1999 matrix theory book.

The first clue that something is wrong is that $\lambda_n(B)$ for an arbitrary complex matrix $B$ may not be a real number, and so the inequality as a whole doesn't make any sense.

After going through the proof of Theorem 7.11 in Zhang's book (which is referenced in the paper you linked for this inequality), I believe that $\lambda_n(B)$ actually means $\lambda_n(\mathcal{H}(B))$, where $$\begin{align} \mathcal{H}(B) &= \begin{bmatrix} 0 & B\\ B^* & 0 \end{bmatrix} \end{align}$$ whence $\lambda_n(\mathcal{H}(B)) = -\sigma_1(B)$, and so the desired inequality actually reads $$ \sigma_j(A + B) \ge \sigma_j(A) - \sigma_1(B) $$

New contributor
AlkaKadri is a new contributor to this site. Take care in asking for clarification, commenting, and answering. Check out our Code of Conduct.
$\endgroup$

Your Answer

By clicking “Post Your Answer”, you agree to our terms of service and acknowledge that you have read and understand our privacy policy and code of conduct.

Not the answer you're looking for? Browse other questions tagged or ask your own question.